one more than seven times the difference of a number and 4 is equivalent to twice the difference of the number and 1

Answers

Answer 1

Answer:

the number is 5

Step-by-step explanation:

We have to find the equation to solve the problem.

Let the number be x

lets write one more than seven times the difference of a number and 4  in mathematical form

1. Difference of number and 4 will be written as x - 4

now

2. seven times the difference of a number and 4 will be written as

7*( x - 4)

3. one more than seven times the difference of a number and 4 is expressed as

1 + 7*( x - 4)

_____________________________________________

1 + 7*( x - 4) is equal to twice the difference of the number and 1

4. difference of the number and 1 is x - 1

5.  twice the difference of the number and 1 is 2*(x-1)

Thus,

1 + 7*( x - 4) = 2*(x-1)

=> 1 + 7x - 28 = 2x -2

=> 7x - 27 = 2x - 2

adding 27 both sides

=> 7x - 27 + 27 = 2x - 2 + 27

=> 7x = 2x + 25

subtracting 2x from both sides

7x -2x = 2x + 25 - 2x

=> 5x = 25

dividing both side by 5

5x/5 = 25/5

x = 5

Thus, the number is 5  answer


Related Questions

Using the table given below, find the number of Tourist Resorts per capita in Istanbul. Write your answer as a decimal correctly rounded to 6 places Write your answer above in scientific notation. Example: 2.9x10^-1 OR 5.8E6 City Population Tourist Resorts Beijing 20.035 million 290 Mexico City 21.671 million 348 Istanbul 15.154 million 160 Paris 11.017 million 232

Answers

Answer:

0.0000111.1E-5

Step-by-step explanation:

In Istanbul, the number of resorts per person is ...

  160/(15.154·10^6) = 0.000011 . . . . rounded to 6 places

  = 1.1E-5 . . . . in scientific notation

What values of X make the two expressions below equal?

Answers

Answer:

A

Step-by-step explanation:

they dont come out the same

Solve for x.

2x/3 + 1 = 3

x = 1
x = 2
x = 3
x = 6

Answers

Answer:

The correct answer is x=3 trust me. Im good at math got 1st place on 1v1 equations in my school hope this helped.

Step-by-step explanation:

Answer:

[tex]\boxed {\tt x=3}[/tex]

Step-by-step explanation:

We want to solve for x, therefore we must isolate the variable.

We are given the equation:

[tex]\frac{2x}{3} +1=3[/tex]

1 is being added to 2x/3. The inverse of addition is subtraction. Subtract 1 from both sides of the equation.

[tex]\frac{2x}{3}+1-1=3-1[/tex]

[tex]\frac{2x}{3} =2[/tex]

2x is being divided by 3. The inverse of division is multiplication. Multiply both sides by 3.

[tex]3*\frac{2x}{3} =2*3[/tex]

[tex]2x=2*3[/tex]

[tex]2x=6[/tex]

x is being multiplied by 2. The inverse of multiplication is division. Divide both sides of the equation by 2.

[tex]\frac{2x}{2} =\frac{6}{2}[/tex]

[tex]x=\frac{6}{2}[/tex]

[tex]x=3[/tex]

The solution to the equation is x=3.

I need help this is math

Answers

B,suppose that x=2 y=3 z=4
(2+3)-(-4)=9=2+(3+4) so the answer to your question is x+(y+z)

Answer:

B. x+(y+z)

Step-by-step explanation:

Minus(minus) gives plus(+)

If a house is worth $125,000 and depreciates by 7.5% per year, how much is it worth in two
years?

Answers

Answer:

18750

Step-by-step explanation:

so since it's 2 years we have to double 7.5 which is 15%. Then we multiply 15 x 125,000 which is 18,750

After two years of depreciation the house would be worth approximately $106,953.

To calculate the worth of the house after two years of depreciation, you need to apply the depreciation rate for each year.

After the first year, the value of the house would decrease by 7.5%.

So, the worth of the house after one year is:

125,000 - (7.5% of 125,000)

= 125,000 - (0.075 × 125,000)

= 125,000 - 9,375

= $115,625.

Now, to find the worth of the house after the second year, you need to apply the depreciation rate again:

115,625 - (7.5% of 115,625)

= 115,625 - (0.075 × 115,625)

= 115,625 - 8,671.88

= 106,953.

So, the house would be worth approximately $106,953 after two years of depreciation.

To learn more on Depreciation rate click here:

https://brainly.com/question/31180880

#SPJ4


[tex]1 \frac{3}{4} [/tex]
The square of the number. answer has to be a mixed number ​

Answers

Answer:

[tex]3 \frac{1}{16} [/tex]

Step-by-step explanation:

first lets change the question into proper fraction

[tex]1 \frac{3}{4} = \frac{7}{4} [/tex]

now lets find the square of the number

[tex] { \frac{7}{4} }^{2} = \frac{49}{16} [/tex]

The question says the answer must be a mixed number so lets change it

[tex] \frac{49}{16} = 3 \frac{1}{16} [/tex]

hope u like it ❤️‍‍❤️‍‍❤️‍

for any question comment me

Which type of transformation is shown here?

Answers

That would be a rotation

Answer:

this one is Rotation

Step-by-step explanation:

it's going to flip on the side

Solve by factoring plz show all steps need ASAP

Answers

That show work from me

3x + 5 =11 use the subtraction property of equality

Answers

2 subtract 5 from both sides, cancel out 5 in left. 3x=6 divide 3 from both sides so x=2

Answer: the answer to the equation would be x=2

Step-by-step explanation:

You would use the subtraction property of equality to get ride of the five. Then you would subtract 11 by 5 and get 6. After that you would get the x by itself and divide it by 3. Them you will also divide 6 by 3 And be left with 2.

Hello i need some help!

Answers

Answer:

3×2 + 3×-b

Step-by-step explanation:

[tex]3(2-b)\\= 3\times 2 + 3\times (-b)\\\\= 6 -3b[/tex]

3(2-b) can be rewritten as 3•2 - 3b

In the distributive property you are distributing the number outside the parentheses to the numbers inside.

what is the sum of 0.64 and 0.28

Answers

Answer:

0.92

Here is my work:

    +1

  0.64

+ 0.28

__________

    0.92

(the +1 is the carrying of the one)

Hope this helps!!

The answer would be 0.92 after adding 0.64 and 0.28 and applying the arithmetic operation.

What are arithmetic operations?

The study of numbers and their operations, which are relevant to all other fields of mathematics, is part of the discipline of arithmetic operations. Addition, subtraction, multiplication, and division are the basic operations that make up this system.

It is given that:

The numbers are 0.64 and 0.28

= 0.64 + 0.28

After applying arithmetic operations,

= 0.92

Thus, the answer would be 0.92 after adding 0.64 and 0.28 and applying the arithmetic operation.

Learn more about the arithmetic operations here,

brainly.com/question/25834626

#SPJ2

13/2 divided by 3/4

Answers

Answer: 26/3

Step-by-step explanation: When dividing by a fraction, we can change

the division sign to multiplication and flip the second fraction.

So 13/2 ÷ 3/4 means the same thing as 13/2 · 4/3.

So when dividing by a fraction, we can just multiply

by the reciprocal of that fraction or that fraction flipped.

Next we cross-cancel.

4 and 2 reduce to 2 and 1.

Multiplying across the numerator and denominators we have 26/3.

Solve. 3 x+2y−z=8 2x+2z=−4 x+3y=4 Enter your answer, in the form (x,y,z), in the boxes in simplest terms. ( , , )

Answers

Answer:

x = 1 , y = 1 , z = -3

Step-by-step explanation:

Solve the following system:

{3 x + 2 y - z = 8

2 x + 2 z = -4

x + 3 y = 4

Hint: | Choose an equation and a variable to solve for.

In the third equation, look to solve for x:

{3 x + 2 y - z = 8

2 x + 2 z = -4

x + 3 y = 4

Hint: | Solve for x.

Subtract 3 y from both sides:

{3 x + 2 y - z = 8

2 x + 2 z = -4

x = 4 - 3 y

Hint: | Perform a substitution.

Substitute x = 4 - 3 y into the first and second equations:

{3 (4 - 3 y) + 2 y - z = 8

2 (4 - 3 y) + 2 z = -4

x = 4 - 3 y

Hint: | Expand the left hand side of the equation 3 (4 - 3 y) + 2 y - z = 8.

3 (4 - 3 y) + 2 y - z = (12 - 9 y) + 2 y - z = 12 - 7 y - z:

{12 - 7 y - z = 8

2 (4 - 3 y) + 2 z = -4

x = 4 - 3 y

Hint: | Expand the left hand side of the equation 2 (4 - 3 y) + 2 z = -4.

2 (4 - 3 y) + 2 z = (8 - 6 y) + 2 z = 8 - 6 y + 2 z:

{12 - 7 y - z = 8

8 - 6 y + 2 z = -4

x = 4 - 3 y

Hint: | Choose an equation and a variable to solve for.

In the first equation, look to solve for y:

{12 - 7 y - z = 8

8 - 6 y + 2 z = -4

x = 4 - 3 y

Hint: | Isolate terms with y to the left hand side.

Subtract 12 - z from both sides:

{-7 y = z - 4

8 - 6 y + 2 z = -4

x = 4 - 3 y

Hint: | Solve for y.

Divide both sides by -7:

{y = 4/7 - z/7

8 - 6 y + 2 z = -4

x = 4 - 3 y

Hint: | Perform a substitution.

Substitute y = 4/7 - z/7 into the second equation:

{y = 4/7 - z/7

8 - 6 (4/7 - z/7) + 2 z = -4

x = 4 - 3 y

Hint: | Expand the left hand side of the equation 8 - 6 (4/7 - z/7) + 2 z = -4.

8 - 6 (4/7 - z/7) + 2 z = 2 z + ((6 z)/7 - 24/7) + 8 = (20 z)/7 + 32/7:

{y = 4/7 - z/7

(20 z)/7 + 32/7 = -4

x = 4 - 3 y

Hint: | Choose an equation and a variable to solve for.

In the second equation, look to solve for z:

{y = 4/7 - z/7

(20 z)/7 + 32/7 = -4

x = 4 - 3 y

Hint: | Isolate terms with z to the left hand side.

Subtract 32/7 from both sides:

{y = 4/7 - z/7

(20 z)/7 = -60/7

x = 4 - 3 y

Hint: | Solve for z.

Multiply both sides by 7/20:

{y = 4/7 - z/7

z = -3

x = 4 - 3 y

Hint: | Perform a back substitution.

Substitute z = -3 into the first equation:

{y = 1

z = -3

x = 4 - 3 y

Hint: | Perform a back substitution.

Substitute y = 1 into the third equation:

{y = 1

z = -3

x = 1

Hint: | Sort results.

Collect results in alphabetical order:

Answer: {x = 1 , y = 1 , z = -3

The quotient of 4 and five times t

Answers

Answer:

[tex]\frac{4}{5t}[/tex]

Step-by-step explanation:

Quotient means division, and 5 times t is 5t, as we are simplifying. Since 4 comes first in the question or sentence, 4 must be the nominator; therefore, 5t is the denominator.

The answer would then be [tex]\frac{4}{5t}[/tex].

Hope this helped!

The diagonals of a rhombus are 4 inches and 18 inches. How long are the sides?

Answers

4x8=842 so that is the answer and here

5) A data set with whole numbers has a low value 20 and high value of 82. Find the class width and class limits for a frequency table with 7 classes.

Answers

Answer:

Class width = 9

Class limits = 20 - 28; 20 - 28; 29 - 37; 38 - 46; 47 - 55; 56 - 64; 65 - 73; 74 - 82.

Step-by-step explanation:

We are given;

Highest value = 82

Lowest value = 20

Frequency = 7

Formula for class width is;

Class width = (Highest Value - Lowest Value)/number of classes

Class width = (82 - 20)/7

Class width = 8.8 ≈ 9

Now,since class width is 9, limits between 20 and 82 would be done at intervals of 9.

Thus;

Class limits = 20 - 28; 20 - 28; 29 - 37; 38 - 46; 47 - 55; 56 - 64; 65 - 73; 74 - 82.

The sale price of a swimming pool after a 18.5% discount is $1092.10. Find the original list price of the pool.

Answers

X-.165x=1210.75
.835x=1210.75 but where did i get .835 from?
.835 .835
X=1450

In the figure above, triangle ABC is inscribed in the circle with center O and diameter AC. If AB=AO, what is the degree measure of∠ABO?
(A) 15°
(B) 30°
(C) 45°
(D) 60°​

Answers

(B) 30 degrees...........

Calculus Ch. 1.2 Classwork Problems Evaluating limits Graphically

Answers

Answer:

8) 1

9) -4

10)  -3

11)  -1

12)  1

13)  doesn't exist

14)  1

15)  doesn't exist

Step-by-step explanation:

8) when we approach x=-8 from left and from right, the function tends towards the value 1

9) when we approach x = -7, from the left the function gets towards 0, while when we approach it from the right it gets closer to -4

10) f(-3) = -3

11) When x approaches the value 4 from the left and from the right, f(x) gets closer to -1

12) f(4) is defined as 1

13) f(6) doesn't exist

14)  When x approaches 6 from the left and from the right, the function approaches 1

15)  When x approaches the value 7 from the left the function gets closer to 2, while when we approach x = 7 from the right the function gets toward 7. Because of this discrepancy, the limit doesn't exist.

I NEED HELP!!!!!! 4x+2x-5=7x-1

Answers

Let's solve your equation step-by-step.

4x+2x−5=7x−1

Step 1: Simplify both sides of the equation.

4x+2x−5=7x−1

4x+2x+−5=7x+−1

(4x+2x)+(−5)=7x−1(Combine Like Terms)

6x+−5=7x−1

6x−5=7x−1

Step 2: Subtract 7x from both sides.

6x−5−7x=7x−1−7x

−x−5=−1

Step 3: Add 5 to both sides.

−x−5+5=−1+5

−x=4

Step 4: Divide both sides by -1.

−x

−1

=

4

−1

x=−4

Answer:

x=−4

i hope this helps you

what is the answer for -9+x=-26

Answers

x=-17 is the answer

Answer:

x =-17

Step-by-step explanation:

[tex]-9+x=-26\\\\\mathrm{Add\:}9\mathrm{\:to\:both\:sides}\\-9+x+9=-26+9\\\\Simplify\\x=-17[/tex]

If 26.97 ÷ 6.3 is written in long division form as long division set up where 63 is on the outside of the division symbol and 26970 is on the inside so that the divisor is written as a whole number, where should the decimal point be placed in the dividend?
Between the 2 and the 6
Between the 6 and the 9
Between the 9 and the 7
Between the 7 and the 0

Answers

The decimal point should be placed between digits 9 and 7 in the dividend. i.e the dividend becomes 269.7

Step-by-step explanation:

Dividend = 26.97       [numerator]

Divisor = 6.3               [denominator]

If 26.97 ÷ 6.3 is written in long division form so that the divisor is written as a whole number, we have the following;

(i)First convert the divisor to a whole number by multiplying by 10 i.e

6.3 x 10 = 63

(ii) Since the divisor (denominator) has been multiplied by 10, to make sure the division expression stays the same, we need to multiply the dividend(numerator) too by 10. i.e

26.97 x 10 = 269.7

(iii) The division expression then becomes;

269.7 ÷ 63

Therefore, the decimal point should be placed between digits 9 and 7 in the dividend.

can you give me Brainliest

Answer:

9 and 7

Step-by-step explanation:

26.6.3 is long division set up 63 is out of the division symbol and 26970.

Please help thank youuu

Answers

Answer:

D: 61 degrees.

Step-by-step explanation:

right angles add up to 90 degrees.

Answer:

61 degrees

Step-by-step explanation:

29+x=90

x=90-29

x=61

-9 would be a part of which group(s)? *
Natural
Whole
Integer
Rational
What is the answer

Answers

Answer:

Integer, Rational

Step-by-step explanation:

Natural numbers are the basic counting numbers (1,2,3...) and does not include negatives.

Whole numbers are natural numbers with the addition of zero, and still does not include negatives.

Integers are all whole numbers and their negative counterparts, which includes -9.

Rational numbers are numbers that can be expressed as a fraction and -9 can be expresses as -9/1, so -9 is a rational number.

Hope this helps!

integer and rational

Is -5/13 a rational number

Answers

Answer:

yes -5/13 is a rational number

5/6 equals 1/3 plus d what does d equal

Answers

It equals 1 1/6 + d.

In other words, one and one sixth plus d.

Solve 3[-x + (2 x + 1)] = X-1.
X =

Answers

X= -2

Yeasssssssssssssss

Answer:

-2

Step-by-step explanation:

took the test

What are the features of the quadratic function graphed in the figure? Question 14 options: A) Vertex = (–4,3), y-intercept = (5,0), x-intercepts = (0,1) and (0,5), axis of symmetry is x = –4 B) Vertex = (3,–4), y-intercept = (0,5), x-intercepts = (1,0) and (5,0), axis of symmetry is x = 3 C) Vertex = (3,–4), y-intercepts = (1,0) and (5,0), x-intercept = (0,5), axis of symmetry is x = 3 D) Vertex = (–3,4), y-intercept = (0,5), x-intercepts = (1,0) and (5,0), axis of symmetry is x = –3

Answers

Answer:

B

Step-by-step explanation:

The vertex is the bottom-most point of the curve located at the halfway point. The x coordinate of the point is 3, and the y coordinate is -4. The places at which the quadratic hits the x axis are at (1,0) and (5,0), and it hits the y axis at (0,5).

Vertex = (3,–4), y-intercept = (0,5), x-intercepts = (1,0) and (5,0), axis of symmetry is x = 3. Therefore, option B is the correct answer.

We need to find the features of the quadratic function graphed in the figure.

What is a vertex in the graph?

"Vertex" is a synonym for a node of a graph, i.e., one of the points on which the graph is defined and which may be connected by graph edges.

The features of the quadratic function are as follows:

The vertex is the bottom-most point of the curve located at the halfway point. The x -coordinate of the point is 3, and the y -coordinate is -4. The places at which the quadratic hits the x-axis are at (1,0) and (5,0), and it hits the y-axis at (0,5).

Therefore, option B is the correct answer.

To learn more about the quadratic function visit:

https://brainly.com/question/27958964.

#SPJ2

Rate of change of rational functions

Answers

Answer:

0.635

Step-by-step explanation:

[tex]rate~of~change = \dfrac{f(x_2) - f(x1)}{x_2 - x_1}[/tex]

[tex]rate~of~change = \dfrac{f(x_2) - f(x1)}{x_2 - x_1}[/tex]

[tex] x_1 = 0 [/tex]       [tex]x_2 = 0.3[/tex]

[tex] f(x) = \dfrac{-7x^3}{x^4 - 1} [/tex]

[tex] f(x_1) = f(0) = \dfrac{-7(0)^3}{(0)^4 - 1} [/tex]

[tex] f(x_1) = \dfrac{0}{-1} = 0 [/tex]

[tex] f(x_2) = f(0.3) = \dfrac{-7(0.3)^3}{(0.3)^4 - 1} [/tex]

[tex]f(x_2) = \dfrac{-0.189}{-0.9919} = 0.190543[/tex]

[tex] rate~of~change = \dfrac{0.190543 - 0}{0.3 - 0} = 0.635 [/tex]

A group incoming first-year students at Major University were surveyed in order to determine the factors that influenced their decisions to choose to attend Major U. The survey revealed the following information. 55 said "great football team" (F) 51 said "excellent party reputation" (P) 61 said "it was the only place that would accept me" (O) 9 said "great football team" but didn't say "excellent party reputation" and didn't say "it was the only place that would accept me" 26 said "it was the only place that would accept me" and "great football team" and "excellent party reputation" 31 said "excellent party reputation" and "great football team" 8 said only "excellent party reputation" 4 said none of the above reasons Required:a. How many said "it was the only place that would accept me" and "excellent party reputation"? b. How many said "great football team "or "it was the only place that would accept me"? c. How many said "great football team "but no "excellent party reputation"? d. How many were surveyed?

Answers

Answer:

Step-by-step explanation:

This is a problem of SETS.

Start by listing out important data:

1. Total that said F = 55

2. Total that said P = 51

3. Total that said O = 61

4. F only = 9

5. F ∩ P ∩ O = 26 [NOTE: If you were to draw a Venn Diagram, 26 would be in the innermost circle because it comprises all three categories]

6. F ∩ P = 31

7. P only = 8

8. Students that said none of the 3 reasons = 4

QUESTIONS

1. How many said O and P? In other words, find the intersect of O and P. Find O ∩ P

2. How many said either F or O? [Answer to be gotten using a venn diagram] Find F ∪ P which translates to "F union P"

3. How many said F without saying P? [Answer to be gotten from the venn diagram as well]

4. How many students in total were surveyed? [HINT: Remember to include the 4 students that had none of the three options]

A) 12 said "it was the only place that would accept me" and "excellent party reputation"; B) 17 said "great football team" or "it was the only place that would accept me"; C) 24 said "great football team" but no "excellent party reputation"; and D) 87 students were surveyed.

Given that a group incoming first-year students at Major University were surveyed in order to determine the factors that influenced their decisions to choose to attend Major U, and the survey revealed that 55 said "great football team" (F), 51 said " excellent party reputation "(P), 61 said" it was the only place that would accept me "(O), 9 said" great football team "but didn't say" excellent party reputation "and didn't say" it was the only place that would accept me ", 26 said" it was the only place that would accept me "and" great football team "and" excellent party reputation ", 31 said" excellent party reputation "and" great football team ", 8 said only "excellent party reputation", and 4 said none of the above reasons, to determine:

A) How many said "it was the only place that would accept me" and "excellent party reputation"?

B) How many said "great football team" or "it was the only place that would accept me"?

C) How many said "great football team" but no "excellent party reputation"?

D) How many were surveyed?

The following calculations must be performed:

F = 55 (F = 9 + FP = 5 + FO = 15 + FPO = 26) = 55 - (26 + 9) = 20 - (31 - 26) = 15 = 9 + 5 + 15 + 26 = 55 // / FO = 15 /// FP = 5 P = 51 (P = 8 + PF = 5 + PO = 12 + PFO = 26) = 51 - 8 - 5 - 26 = 12 /// PO = 12 O = 61 (O = 8 + OP = 12 + OF = 15 + OPF = 26) = 61 - 12 - 15 - 26 = O = 8 None = 4  4 + 9 + 8 + 8 + 5 + 12 + 15 + 26 = X 87 = X

Therefore, A) 12 said "it was the only place that would accept me" and "excellent party reputation"; B) 17 said "great football team" or "it was the only place that would accept me"; C) 24 said "great football team" but no "excellent party reputation"; and D) 87 students were surveyed.

Learn more about surveys in https://brainly.com/question/21602839

Other Questions
Why were Europeans eagerly searching for a sea route to Asia in the fifteenth century Based on latitude, which city would be colder than Dallas? Somebody please help!!!(Dont judge me I forgot) what is 23/11 in decimal form with an explanation Which is a long-term consequence of an injury? Read the passage below. Then explain how the use of a third-person omniscient point of view helps develop the characters in the story. Use specific examples from the text in supporting your response.PassageAs the credits faded out, Milo pressed stop on the remote control and turned to his best friends, Phil, Britt, and Clay, who were jammed into the small love seat in front of the TV. Milo was excited to hear their thoughts on his first short film but was nervous just the same. Phil held his tongue, even though he wanted to tell Milo that clearly Milo never should have left medical school for this. Britt kept thinking that this was the worst movie she'd ever seen, like it was made in 10 minutes on someone's smartphone. Clay had drifted off halfway through and had become more concerned by a bird that was flapping outside the basement window. In the silence, Milo finally spoke: "Well, what did you think? Should I move to Hollywood?" Milo's three friends shot up and applauded, yelling "Bravo!" and whistling as they showered Milo with compliments. Milo beamed with pride. What is the year 1987 in latin? Find the area of each Who is most likely to get a torn ACL?A 26 year old male basketball playerB 33 year old female runnerC 78 year male with a low activity rateD 40 year old male who has torn his left ACL in the past and has moderate activity rate A triangle has sides of length 4 inches and 7.5 inches. Which of thefollowing CANNOT be the length of the third side?F. 3.5 inchesG. 4.0 inchesH. 5.0 inchesJ. 5.5 inchesK. 9.5 inches When carrying your tablet from one room to the next, it is best to carry it under your arm put it in your pocket use one hand use two hands Circle the letter for the correct answer.7 Last summer, we drove cross-country. The road trip was a totalof 6,765 miles. If we go the samedistance each day for 33 days,what was the average number ofmiles we drove each day? Need done ASAPAfter measuring the foot size of 15 of his biology classmates, Laquon finds the average value and claims that it is theaverage foot size of all 1,200 students in his high school.What is most likely a problem with Laquon's conclusion?A) He should have found the mode instead of the average.B) He should have used shoe size instead of measuring feet.C) The amount of data he collected was inadequate.D) The data he collected was measured incorrectly. At which latitude is Antartica? Georgina bought 6 shirts that costthe same amount of money. Shehad a $15 off coupon for theshirts. Using the coupon,Georgina spent $39. Find the costof each shirt before the couponwas applied. (Don't forget yourunits!) What is the volume of an object that has a mass of 18 g and a density of 0.5 g/cm3 ? In a fluorescence recovery after photobleaching (FRAP) experiment, a fluorescently tagged membrane protein, A, shows very little recovery of fluorescence ten minutes after photobleaching, while membrane protein B shows a rapid increase in fluorescence after bleaching, recovering nearly 80% of its original fluorescent signal by ten minutes. Based on this information, which of the following statements can be made?A. Proteins A and B interact with each other in a complex B. Protein A is easily denatured, C. Protein Bis diffusing in a more fluid membrane. D. Protein Ahas faster mobility in the membrane. PLEASE HELP ASAP!!!!!!!!!!! WITH PAGE 2 FOR QUESTION # 3 WITH THE GRAPHING THANK YOU. THE QUESTION IS AS FOLLOWED: GRAPH THE VAPOR PRESSURE OF WATER USING THE FOLLOWING TEMPERATURES. Temperature (k) Vapor Pressure (Torr) (dependent variable) 273 4.6 293 17.5 313 55.0 333 149.2 353 355.5 373 760.0 Trepaba a ________ rbol del jardn.A) aquellaTrepaba a ________ rboles del jardn.B) esteVolabas ________ cometa del nio.C) aquellasVolabas ________ cometas del nio.D) estos What is the value of the product (3 2i)(3 + 2i)? 5 9 + 4i 9 4i 13